Beware of the Integrable Trap: A Surprising Limit to Convergence

Here is a persuasive advertisement for ghostwriting services specifically targeting students in need of help with uniform convergence and pointwise convergence of sequences of functions:

**Unlock the Secrets of Convergence with Our Expert Ghostwriting Services**

Are you struggling to grasp the complex concepts of uniform convergence and pointwise convergence of sequences of functions? Do you need help with assignments, papers, or projects that involve these topics?

Our team of expert ghostwriters is here to help! With years of experience in mathematics and academic writing, we can assist you in producing high-quality content that meets your academic needs.

**Our Services:**

* Customized assignments and papers on uniform convergence and pointwise convergence
* In-depth explanations and illustrations to help you understand the concepts
* Error-free and plagiarism-free writing that meets your academic standards
* Timely delivery to meet your deadlines

**Our Advantages:**

* Expertise in mathematics and academic writing
* Fast turnaround times without compromising on quality
* Confidential and secure services to protect your academic integrity
* Affordable prices without compromising on quality

**Why Choose Us?**

* Get high-quality content that meets your academic standards
* Save time and effort by letting us handle the writing
* Improve your understanding of uniform convergence and pointwise convergence with our expert explanations
* Boost your academic performance with our help

Don’t let convergence concepts hold you back any longer. Contact us today to get the help you need to succeed!

Since \(d_{n}=\sup\left\{\left|\dfrac{x}{1+nx}\right|:x\in[0,1],n\in\mathbb{N}\right\}\), choose \(d_{n}=\dfrac{1}{1+n}\) which goes to \(0\) as \(n\to\infty\). Therefore, we have uniform convergence.

**Example 83**.: When a sequence of integrable functions \(\{f_{n}(x)\}\) converges pointwise to \(f(x)\) on an interval \([a,b]\), the limit function \(f\) need not to be integrable (Figure 2.14). For example, take \(f_{n}(x)=ng_{n}(x)\) where \(g_{n}(x)\) is the “triangle function” defined as:

\[g_{n}(x)=\begin{cases}nx&\text{ if }x\in[0,1/n]\\ 2-nx&\text{ if }x\in[1/n,2/n]\\ 0&\text{ if }x\in[2/n,1]\end{cases}.\]

The sequence \(\{f_{n}\}\) converges pointwise (but not uniformly) to \(f\equiv 0\), yet

\[\int_{0}^{1}f_{n}(x)\,dx=1,\ \text{ for every n, whereas }\int_{0}^{1}f(x)\,dx=0.\]

The next theorem shows that integrals behave nicely under uniform convergence.

**Theorem 94** (Interchange of limit and integral).: _If the functions \(f_{1},f_{2},\dots\) are integrable over an interval \([a,b]\) and if \(f_{n}\rightrightarrows f\) (uniformly) on \([a,b]\), then \(f\) is integrable and_

\[\int_{a}^{b}f(x)\,dx=\lim_{n\to\infty}\int_{a}^{b}f_{n}(x)\,dx.\]

Proof

: We again appeal to uniform convergence. Thus for all \(\epsilon>0\), there exists \(N\in\mathbb{N}\) such that for all \(n>N\), \(|f_{n}(x)-f(x)|<\epsilon\). So we have

\[\left|\int_{a}^{b}f_{n}-\int_{a}^{b}f\right|=\left|\int_{a}^{b}f_{n}-f\right| \leq\int_{a}^{b}|f_{n}-f|<\epsilon(b-a).\]

It remains to show that \(f\) is integrable. We need to show \(U(f,P)-L(f,P)<\epsilon\) where \(U(f,P)\) and \(L(f,P)\) are upper and lower sums as used in the definition of the Riemann integral. However, for any partition \(P\) of \([a,b]\),

\[U(f,P)-L(f,P) \leq |U(f,P)-U(f_{n},P)|+|U(f_{n},P)-L(f_{n},P)|\] \[+|L(f_{n},P)-L(f,P)|\]

holds true. Let \(\epsilon>0\), then by uniform convergence there is an \(N\) such that the inequalities

\[|U(f,P)-U(f_{n},P)|<\frac{\epsilon}{3}\ \ \text{and}\ \ |L(f_{n},P)-L(f,P)|< \frac{\epsilon}{3}\]

hold true for all \(n\geq N\) and for all partitions \(P\) of \([a,b]\). On the other hand, integrability of \(f_{n}\) allows for a partition P for which \(|U(f_{n},P)-L(f_{n},P)|<\frac{\epsilon}{3}\).

To see how uniform convergence behaves with respect to differentiation, consider the sequence of functions \(f_{n}(x)=|x|^{1+\frac{1}{n}}\). Note that \(f_{n}(x)\to f(x)=|x|\) and that \(f_{n}\rightrightarrows f\) (uniformly) on \([-1,1]\). Even though each \(f_{n}\) is differentiable, \(f\) is not differentiable at zero. Thus while continuity is preserved, differentiability is not preserved under uniform convergence. Another common example is the sequence of functions \(f_{n}(x)=\frac{1}{n}\sin(n^{2}x)\). These functions are differentiable and converge uniformly to \(f(x)\equiv 0\), but their derivatives \(f_{n}^{{}^{\prime}}(x)=n\cos(n^{2}x)\) do not even stay bounded if we take their limit as \(n\to\infty\).

**Theorem 95** (Limit of a derivative is the derivative of the limit).: _Suppose \(\{f_{n}\}\) is a sequence of continuous functions on an interval \([a,b]\) which have continuous derivatives on \((a,b)\). Assume that \(f_{n}\to f\) for each \(x\in[a,b]\) and the sequence of derivatives \(\dot{f_{n}^{{}^{\prime}}}\rightrightarrows g\) on \((a,b)\). Then the limit function \(f\) is differentiable on \((a,b)\) and \(g=f^{\prime}\)._

Proof

: Since \(f_{n}^{{}^{\prime}}\rightrightarrows g\) and each \(f_{n}^{{}^{\prime}}(x)\) is continuous we know that \(g(x)\) is a continuous function on \((a,b)\). Furthermore, uniform continuity allows us to interchange the limit and integration, thus

\[\int_{a}^{x}g(t)dt=\lim_{n\to\infty}\int_{a}^{x}f_{n}^{{}^{\prime}}(t)dt= \lim_{n\to\infty}[f_{n}(x)-f_{n}(a)]=f(x)-f(a)\]

holds true for each \(x\in(a,b)\). Thus, by the Fundamental Theorem of Calculus, \(f\) is differentiable and \(f^{\prime}(x)=g(x)\) on \((a,b)\)

### 2.3 Modes of Convergence

**Corollary 16**.: _If the sequence of functions \(g_{1},g_{2},\dots\) are integrable over an interval \([a,b]\) and the infinite series \(\sum_{n=1}^{\infty}g_{n}\rightrightarrows g\) on \([a,b]\), then \(g\) is integrable and_

\[\int_{a}^{b}g(x)\,dx=\sum_{n=1}^{\infty}\int_{a}^{b}g_{n}(x)\,dx.\]

This result is expressed roughly by indicating that a uniformly convergent series of functions may be integrated term by term, or that summing a series and then integrating the sum is equal to integrating each term and then summing up the integrals.

### Weierstrass M-test

The following is a very useful test by which a series of functions may sometimes be shown to be uniformly convergent.

**Theorem 97** (Weierstrass M-test).: _Let \(\{f_{k}\}\) be a sequence of functions with domain \(A\) and let there exist constants \(M_{k}\) with_

\[|f_{k}(x)|\leq M_{k}\]

_for all \(x\in A\) and \(\sum_{k=1}^{\infty}M_{k}\) convergent. Then the series \(\sum_{k=1}^{\infty}f_{k}\) converges uniformly and absolutely._

Proof

: Since we have \(|f_{k}(x)|\leq M_{k}\) for all \(x\in A\), we know that

\[\sum_{k=1}^{\infty}|f_{k}(x)|\leq\sum_{k=1}^{\infty}M_{k}.\]

So by comparison test this series converges absolutely and therefore it converges for each \(x\in A\). Thus, the series \(\sum f_{k}(x)\) converges pointwise to a function which we will call \(s(x)\). Then we look at \(s_{n}=f_{1}+f_{2}+\dots+f_{n}\) and \(t_{n}=M_{1}+M_{2}+\dots+M_{n}\) where each \(s_{n}\) is a function and each \(t_{n}\) is a real number. Say that \(s_{n}\to s\) and \(t_{n}\to t\). Thus, for each \(x\), if \(n>m\) we have

\[|s_{n}(x)-s_{m}(x)|=\left|\sum_{k=m+1}^{n}f_{k}(x)\right|\leq\sum_{k=m+1}^{n} |f_{k}(x)|<\sum_{k=m+1}^{n}M_{k}=t_{n}-t_{m}.\]

Thus we have that the sequence

\[\{|s_{n}(x)-s_{m}(x)|\}_{n=1}^{\infty}\]

and \(\{t_{n}-t_{m}\}\) are both convergent. If we take the limit as \(n\to\infty\), then we know that \(|s(x)-s_{m}(x)|\leq t-t_{m}\) for all \(m\in\mathbb{N}\) and \(x\in A\). When \(m\to\infty\), \(t-t_{m}\to 0\), and this gives the uniform convergence.

**Example 84**.:
1. The series \(\sum_{n=1}^{\infty}\frac{(\sin(nx))^{2}}{n^{2}}\) converges uniformly. This can be deduced by considering the inequality: \[\left|\frac{(\sin(nx))^{2}}{n^{2}}\right|\leq\frac{1}{n^{2}}.\] Let \(M_{n}=\frac{1}{n^{2}}\); since we know that \(\sum\frac{1}{n^{2}}\) is a convergent \(p\)-series (with \(p=2\)), by the Weierstrass M-test, the given series converges uniformly.
2. The series \(\sum_{n=1}^{\infty}e^{-nx}\) converges uniformly on any closed subinterval of \((0,\infty)\). Let \(I=[a,\infty)\). If \(x\in I\), then \(e^{-nx}\leq e^{-na}\). We know that \(\sum_{n=0}^{\infty}e^{-na}\) is a convergent geometric series. Therefore, by the Weierstrass M-test, the series converges uniformly.

**Remark 48**.: A simple and useful test for uniform convergence called _Dirichlet’s test_ is as follows: Suppose that the \(N\)-th partial sum of the series \(\sum f_{n}(x)\) is uniformly bounded with respect to both \(x\) and \(N\) on the interval \(I\) and that \(g_{n}(x)\) is a monotone decreasing sequence converging uniformly to zero. Then the series \(\sum_{n=1}^{\infty}f_{n}(x)g_{n}(x)\) converges uniformly on \(I\). (see, e.g., [26], p. 287, also Exercise 11 below).

The Cauchy criterion for convergence of a numerical sequence can be adapted to a criterion for uniform convergence.

**Definition 77** (Uniform Cauchy Criterion).: Suppose \(\{f_{n}\}\) is a sequence of functions defined on \(D\subset\mathbb{R}\). Then we say the sequence \(f_{n}\) is a _uniform Cauchy sequence_ on \(D\) if for every \(\epsilon>0\), there exists \(N\in\mathbb{N}\) such that \(n,m>N\) implies that

\[|f_{n}(x)-f_{m}(x)|<\epsilon\]

for all \(x\in D\).

It is not hard to observe that every uniform Cauchy sequence is uniformly convergent. To see this first apply the uniform Cauchy property to state

\[|f_{n}(x)-f_{m}(x)|<\epsilon\ \ \text{for all}\ \ x\in D\]

when \(n,m\geq N.\) Now, fix \(n\) and take limit as \(m\to\infty\). Since \(f_{m}\to f\) as \(m\to\infty\), we obtain that

\[|f_{n}(x)-f(x)|<\epsilon\]

for all \(x\in D\) which implies that \(f_{n}\rightrightarrows f\) uniformly. Note that the Weierstrass M-test can be obtained as an application of the uniform Cauchy criterion.

**Remark 49**.: In some of the above proofs that involve integrals we use the following inequalities many times. Perhaps it is useful to list them:

1. If \(f\) and \(g\) are integrable functions on \([a,b]\), and \(f(x)\leq g(x)\) for all \(x\in[a,b]\), then \[\int_{a}^{b}f(x)dx\leq\int_{a}^{b}g(x)dx.\]
2. If \(f\) is integrable on \([a,b]\), so is the function \(|f|\), and \[\left|\int_{a}^{b}f(x)dx\right|\leq\int_{a}^{b}|f(x)|dx.\] The above inequality (b) is the integral version of the well known triangle inequality \(|a+b|\leq|a|+|b|\) for any numbers \(a\) and \(b\). If one assumes the integrability of \(|f|\), then (b) follows from (a) and from the fact that \[-|f(x)|\leq f(x)\leq|f(x)|.\]

**Remark 50**.: There are many excellent texts in analysis and many contain the topics of uniform and pointwise convergence and their consequences. For example [1, 18, 26, 42, 52] are such texts.

uniform convergence

评论

发表回复

您的电子邮箱地址不会被公开。 必填项已用 * 标注